Lorentz Transformations boost?

It looks like the map from Minkowski space to this vector space is just given by the Lorentz transformation, but I'm not sure how to show that B(\theta, \vec{n}) corresponds to a Lorentz boost with velocity \vec{v}=\tanh{\theta}\vec{n}. In summary, B(\theta, \vec{n}) is defined as a matrix in SL(2, \mathbb{C}) and has a specific form involving the hyperbolic cosine and sine functions. It can be shown that this corresponds to a Lorentz boost with velocity \vec{v}=\tanh{\theta}\vec{n}. Further, it can be shown that the matrix B(\theta, \
  • #1
latentcorpse
1,444
0
Define [itex]B( \theta, \vec{n} ) \in SL( 2 , \mathbb{C} )[/itex] by

[itex]B( \theta , \vec{n}) = \cosh { \frac{1}{2} \theta} + \vec{\sigma} \cdot \vec{n} \sinh{ \frac{1}{2} \theta}[/itex] where [itex]\vec{n}^2 =1 [/itex]

Show that this corresponds to a Lorentz boost with velocity [itex]\vec{v}=\tanh{ \theta} \vec{n}[/itex]. Show that

[itex] ( 1 + \frac{1}{2} \vec{sigma} \cdot \delta \vec{v}) B(\theta, \vec{n}) = B( \theta' , \vec{n}' ) R[/itex]

where, to 1st order in [itex]\delta \vec{v}[/itex],

[itex]\theta' = \theta + \delta \vec{v} \cdot \vec{n}[/itex], [itex]\vec{n}'=\vec{n} \coth{\theta} (\delta \vec{v} - \vec{n} \vec{n} \cdot \delta \vec{v})[/itex]

and [itex]R[/itex] is an infinitesimal rotation given by

[itex]R= 1 + \tanh{\frac{1}{2} \theta} \frac{1}{2}i ( \delta \vec{v} \times \vec{n} ) \cdot \vec{\sigma} = 1 + \frac{\gamma}{\gamma + 1} \frac{1}{2} i ( \delta \vec{v} \times \vec{v} ) \cdot \vec{\sigma}[/itex]
and [itex]\gamma = ( 1 - \vec{v}^2) ^{-\frac{1}{2}}[/itex]

Show that [itex]\vec{v}' = \vec{v} + \delta \vec{v} - \vec{v} \vec{v} \cdot \delta \vec{v}[/itex]

Note that [itex]\vec{\sigma} \cdot \vec{a} \vec{\sigma} \cdot \vec{b} = \vec{a} \cdot \vec{b} 1 + i \vec{\sigma} \cdot ( \vec{a} \times \vec{b} )[/itex]I don't understand how to go about the first bit here - what do I need to do in order to show this is a Lorentz boost?

Thanks!
 
Physics news on Phys.org
  • #2


latentcorpse said:
I don't understand how to go about the first bit here - what do I need to do in order to show this is a Lorentz boost?

Thanks!

[tex]B( \theta, \vec{n} )[/tex] is an element of a representation of [tex]SL( 2 , \mathbb{C} )[/tex]. What vector space does the representation act on? Is there a map from Minkowski space to this vector space that you know of?
 
  • #3


fzero said:
[tex]B( \theta, \vec{n} )[/tex] is an element of a representation of [tex]SL( 2 , \mathbb{C} )[/tex]. What vector space does the representation act on? Is there a map from Minkowski space to this vector space that you know of?

well [itex]SL(2, \mathbb{C})[/itex] is just matrices so surely they can act on any vector in [itex]\mathbb{R}^2[/itex]?
 

Related to Lorentz Transformations boost?

1. What are Lorentz Transformations boost?

Lorentz Transformations boost, also known as Lorentz boosts, are mathematical equations that describe the relationship between space and time in special relativity. They are used to convert measurements of space and time from one reference frame to another that is moving at a constant velocity relative to the first frame.

2. Why are Lorentz Transformations boost important?

Lorentz Transformations boost are important because they allow us to understand and predict the effects of time dilation and length contraction in special relativity. They also help us reconcile the differences between measurements made by observers in different reference frames.

3. How do Lorentz Transformations boost work?

Lorentz Transformations boost use a set of equations, including the famous equation E=mc2, to convert measurements of space and time between two reference frames. They take into account the relative velocity between the two frames and the speed of light, which is constant in all frames.

4. Are Lorentz Transformations boost only applicable to high speeds?

No, Lorentz Transformations boost are applicable to any situation where objects are moving at a constant velocity relative to each other. While they are commonly associated with high speeds and special relativity, they can also be applied to everyday situations such as the movement of cars or airplanes.

5. Can Lorentz Transformations boost be used in all reference frames?

Yes, Lorentz Transformations boost can be used in all inertial reference frames, which are frames that are not accelerating. This includes both inertial frames at rest and those in motion at a constant velocity. However, they cannot be used in non-inertial frames, such as those experiencing acceleration or rotation.

Similar threads

Replies
1
Views
422
  • Advanced Physics Homework Help
Replies
3
Views
443
  • Advanced Physics Homework Help
Replies
2
Views
1K
  • Advanced Physics Homework Help
Replies
1
Views
481
  • Advanced Physics Homework Help
Replies
9
Views
949
  • Advanced Physics Homework Help
Replies
1
Views
714
  • Advanced Physics Homework Help
Replies
3
Views
873
  • Advanced Physics Homework Help
Replies
1
Views
935
  • Advanced Physics Homework Help
Replies
2
Views
773
  • Advanced Physics Homework Help
Replies
9
Views
1K
Back
Top